Nguyễn Thảo Nhi

cho a,b,c >0 va abc=1 c/m

\(\frac{1+ab^2}{c^3}+\frac{1+bc^2}{a^3}+\frac{1+ca^2}{b^3}>=\frac{18}{a^3+b^3+c^3}\)

alibaba nguyễn
21 tháng 9 2016 lúc 21:55

Ta có 1 + ab2 \(\ge\)\(2b\sqrt{a}\)

1 + bc2 \(\ge2c\sqrt{b}\)

1 + ca2 \(\ge2a\sqrt{c}\)

VT \(\ge\)\(2\left(\frac{b\sqrt{a}}{c^3}+\frac{c\sqrt{b}}{a^3}+\frac{a\sqrt{c}}{b^3}\right)\)

\(\ge2\frac{\left(\sqrt[4]{b^2a}+\sqrt[4]{c^2b}+\sqrt[4]{a^2c}\right)^2}{a^3+b^3+c^3}\)

\(\ge2\frac{\left(3\sqrt[12]{a^3b^3c^3}\right)^2}{a^3+b^3+c^3}\)

\(\ge\frac{18}{a^3+b^3+c^3}\)

Bình luận (0)

Các câu hỏi tương tự
Đỗ UYển dương
Xem chi tiết
Itachi Uchiha
Xem chi tiết
Nguyễn Hoài Phương
Xem chi tiết
Pham Van Hung
Xem chi tiết
Nguyễn Duy Long
Xem chi tiết
trần xuân quyến
Xem chi tiết
Kan
Xem chi tiết
Le Dinh Quan
Xem chi tiết
Hà Lê
Xem chi tiết